www.matheraum.de
Das Matheforum.
Das Matheforum des MatheRaum.

Für Schüler, Studenten, Lehrer, Mathematik-Interessierte.
Hallo Gast!einloggen | registrieren ]
Startseite · Forum · Wissen · Kurse · Mitglieder · Team · Impressum
Forenbaum
^ Forenbaum
Status Mathe
  Status Schulmathe
    Status Primarstufe
    Status Mathe Klassen 5-7
    Status Mathe Klassen 8-10
    Status Oberstufenmathe
    Status Mathe-Wettbewerbe
    Status Sonstiges
  Status Hochschulmathe
    Status Uni-Analysis
    Status Uni-Lin. Algebra
    Status Algebra+Zahlentheo.
    Status Diskrete Mathematik
    Status Fachdidaktik
    Status Finanz+Versicherung
    Status Logik+Mengenlehre
    Status Numerik
    Status Uni-Stochastik
    Status Topologie+Geometrie
    Status Uni-Sonstiges
  Status Mathe-Vorkurse
    Status Organisatorisches
    Status Schule
    Status Universität
  Status Mathe-Software
    Status Derive
    Status DynaGeo
    Status FunkyPlot
    Status GeoGebra
    Status LaTeX
    Status Maple
    Status MathCad
    Status Mathematica
    Status Matlab
    Status Maxima
    Status MuPad
    Status Taschenrechner

Gezeigt werden alle Foren bis zur Tiefe 2

Navigation
 Startseite...
 Neuerdings beta neu
 Forum...
 vorwissen...
 vorkurse...
 Werkzeuge...
 Nachhilfevermittlung beta...
 Online-Spiele beta
 Suchen
 Verein...
 Impressum
Das Projekt
Server und Internetanbindung werden durch Spenden finanziert.
Organisiert wird das Projekt von unserem Koordinatorenteam.
Hunderte Mitglieder helfen ehrenamtlich in unseren moderierten Foren.
Anbieter der Seite ist der gemeinnützige Verein "Vorhilfe.de e.V.".
Partnerseiten
Mathe-Seiten:Weitere Fächer:

Open Source FunktionenplotterFunkyPlot: Kostenloser und quelloffener Funktionenplotter für Linux und andere Betriebssysteme
StartseiteMatheForenUni-StochastikMomente
Foren für weitere Schulfächer findest Du auf www.vorhilfe.de z.B. Deutsch • Englisch • Französisch • Latein • Spanisch • Russisch • Griechisch
Forum "Uni-Stochastik" - Momente
Momente < Stochastik < Hochschule < Mathe < Vorhilfe
Ansicht: [ geschachtelt ] | ^ Forum "Uni-Stochastik"  | ^^ Alle Foren  | ^ Forenbaum  | Materialien

Momente: Erwartungswert
Status: (Frage) beantwortet Status 
Datum: 23:13 Mo 29.05.2006
Autor: Karl_Pech

Hallo Zusammen,


Es gibt da noch eine Aufgabe, wo ich nicht weiterkomme. Und zwar ...

Aufgabe

Sei [mm]X[/mm] eine [mm]N\left(0,\sigma^2\right)\texttt{-verteilte}[/mm] Zufallsgröße. Für [mm]k\in\mathbb{N}[/mm] bezeichne [mm]m_k := E\left(X^k\right)[/mm] das [mm]k\texttt{-te}[/mm] Moment von [mm]X[/mm]. Zeigen Sie [mm]m_k = 1\cdot{3}\cdot{}\ldots\cdot{}(k-1)\sigma^k[/mm], falls [mm]k[/mm] gerade ist.



Ich habe nun erstmal die Definition des Erwartungswertes stetiger Zufallsgrößen genommen und versucht einen "Substitutionstrick" anzuwenden, den ich in der Vorlesung gesehen habe( und der für [mm]k[/mm] ungerade ;-) auch funktioniert hat):


[mm]E\left(X^k\right) = \int_{-\infty}^\infty{x^kf(x)\,\mathrm{d}x} = \int_{-\infty}^0{x^kf(x)\,\mathrm{d}x}+\int_0^\infty{x^kf(x)\,\mathrm{d}x}[/mm]


Substituiert man fürs erste Integral [mm]x(z) := -z, x'(z) = -1[/mm], erhält man


[mm]\int_{\overline{z}(-\infty)}^{\overline{z}(0)}{(-z)^kf(-z)(-1)\,\mathrm{d}z} = -\int_{\infty}^0{z^kf(z)\,\mathrm{d}z} = \int_0^{\infty}{z^kf(z)\,\mathrm{d}z}[/mm]


Also gilt:


[mm]E\left(X^k\right) = 2\int_0^{\infty}{x^kf(x)\,\mathrm{d}x}.[/mm]


Nur weiß ich im Moment nicht, ob mir das für die Aussage, die ich beweisen soll, irgendwie nützlich sein könnte. Oder hätte ich hier doch einen anderen Ansatz wählen sollen?


Vielen Dank für die Hilfe!



Grüße
Karl





        
Bezug
Momente: Antwort
Status: (Antwort) fertig Status 
Datum: 23:52 Mo 29.05.2006
Autor: felixf

Hallo Karl!

> Es gibt da noch eine Aufgabe, wo ich nicht weiterkomme. Und
> zwar ...
>  
>
> Sei [mm]X[/mm] eine [mm]N\left(0,\sigma^2\right)\texttt{-verteilte}[/mm]
> Zufallsgröße. Für [mm]k\in\mathbb{N}[/mm] bezeichne [mm]m_k := E\left(X^k\right)[/mm]
> das [mm]k\texttt{-te}[/mm] Moment von [mm]X[/mm]. Zeigen Sie [mm]m_k = 1\cdot{3}\cdot{}\ldots\cdot{}(k-1)\sigma^k[/mm],
> falls [mm]k[/mm] gerade ist.
>  
>
>
> Ich habe nun erstmal die Definition des Erwartungswertes
> stetiger Zufallsgrößen genommen und versucht einen
> "Substitutionstrick" anzuwenden, den ich in der Vorlesung
> gesehen habe( und der für [mm]k[/mm] ungerade ;-) auch funktioniert
> hat):

Versuchs mal lieber mit partieller Integration. Hier hab ich das mal fuer $k = 4$ durchgerechnet und das auf $k = 2$ zurueckgefuehrt.

Per Induktion nach [mm] $\frac{k}{2}$ [/mm] und mit partieller Integration im Induktionsschluss solltest du also weiterkommen...

LG Felix


Bezug
                
Bezug
Momente: Danke
Status: (Mitteilung) Reaktion unnötig Status 
Datum: 15:22 Di 30.05.2006
Autor: Karl_Pech

Hallo Felix!


> Versuchs mal lieber mit partieller Integration.

> Per Induktion nach [mm]\frac{k}{2}[/mm] und mit partieller
> Integration im Induktionsschluss solltest du also
> weiterkommen...


Danke für den Hinweis mit der partiellen Induktion. Damit kann man diese Formel direkt herleiten sogar ohne Induktion, wie ich festgestellt habe.

Zuerst beweisen wir eine nützliche Identität.

Sei dazu [mm]f(x) := \tfrac{1}{\sqrt{2\pi}\sigma}e^{-\frac{x^2}{2\sigma^2}}[/mm]. Dann gilt für die Ableitung nach der Kettenregel:


[mm]f'(x) = \frac{1}{\sqrt{2\pi}\sigma}\left(-\frac{x}{\sigma^2}\right)e^{-\frac{x^2}{2\sigma^2}} = -\frac{x}{\sigma^2}f(x)\quad(\star)[/mm]


Jetzt benutze ich deinen Tipp mit partieller Integration:


[mm]\int_{-\infty}^{\infty}{x^kf(x)\,\mathrm{d}x} \mathop =^{\star} \left.\frac{x^{k+1}}{k+1}f(x)\right|_{-\infty}^{\infty}-\int_{-\infty}^{\infty}{\frac{x^{k+1}}{k+1}\left(-\frac{x}{\sigma^2}\right)f(x)\,\mathrm{d}x}[/mm]


Wegen der Symmetrie von [mm]f[/mm] gilt:


[mm]\left.\frac{x^{k+1}}{k+1}f(x)\right|_{-\infty}^{\infty} = \lim_{z\to\infty}{\left(\frac{z^{k+1}}{k+1}f(z)-\frac{(-z)^{k+1}}{k+1}f(-z)\right)} \mathop =^{k\text{ gerade}} \lim_{z\to\infty}{\left(\frac{z^{k+1}}{k+1}f(z)-\frac{-z^{k+1}}{k+1}f(z)\right)} = \lim_{z\to\infty}{\frac{2z^{k+1}}{k+1}f(z)}[/mm]

[mm]= \lim_{z\to\infty}{\frac{2z^{k+1}}{k+1}\frac{1}{\sqrt{2\pi}\sigma}e^{-\frac{z^2}{2\sigma^2}}} = \sqrt{\frac{2}{\pi}}\frac{1}{(k+1)\sigma}\lim_{z\to\infty}{\frac{z^{k+1}}{e^{z^2/2\sigma^2}}}=0[/mm]


Damit gilt:


[mm]\int_{-\infty}^{\infty}{x^kf(x)\,\mathrm{d}x} = \frac{1}{(k+1)\sigma^2}\int_{-\infty}^{\infty}{x^{k+2}f(x)\,\mathrm{d}x} \gdw m_k =\frac{1}{(k+1)\sigma^2}m_{k+2}[/mm]


Dies ist eine Rekursion, welche jedoch in aufsteigender Form aufgeschrieben ist. Also formen wir sie in eine absteigende Form um:


[mm]m_k =\frac{1}{(k+1)\sigma^2}m_{k+2} \gdw m_{k+2}=(k+1)\sigma^2m_k \gdw m_k=(k-1)\sigma^2m_{k-2}[/mm]


Jetzt "entfalten" wir die Rekursion; Mal sehen was dann passiert... ;-)


[mm]m_k=(k-1)\sigma^2m_{k-2}=(k-1)\sigma^2(k-3)\sigma^2m_{k-4}=(k-1)\sigma^2(k-3)\sigma^2(k-5)\sigma^2m_{k-6} = \dotsb = (k-1)\sigma^2(k-3)\sigma^2(k-5)\sigma^2\dotsc(k-k+1)\sigma^2m_0[/mm]


Und was ist [mm]m_0[/mm]? Das ist per Definition einer Dichtefunktion:


[mm]\int_{-\infty}^{\infty}{x^0f(x)\,\mathrm{d}x} = 1[/mm]


Es bleibt die Frage, aus wievielen Faktoren der Form [mm](k-(2i-1))\sigma^2[/mm] das obige Produkt besteht. Da aber [mm]k[/mm] gerade ist, und jeder Faktor [mm](k-(2i-1))[/mm] eine ungerade Zahl ist, gibt es insgesamt [mm]\tfrac{k}{2}[/mm] solche Faktoren. Damit gilt nach Umstellen der Faktoren:


[mm]m_k = 1\dotsc(k-5)(k-3)(k-1)\sigma^{\frac{2k}{2}}[/mm]


oder anders aufgeschrieben:


[mm]m_k = 1\cdot{3}\dotsc(k-1)\sigma^k.\quad\Box[/mm]



Also danke nochmal für den Tipp.



Grüße
Karl





Bezug
Ansicht: [ geschachtelt ] | ^ Forum "Uni-Stochastik"  | ^^ Alle Foren  | ^ Forenbaum  | Materialien


^ Seitenanfang ^
www.matheforum.net
[ Startseite | Forum | Wissen | Kurse | Mitglieder | Team | Impressum ]